Đến nội dung

lahantaithe99 nội dung

Có 878 mục bởi lahantaithe99 (Tìm giới hạn từ 12-05-2020)



Sắp theo                Sắp xếp  

#493326 Ảnh thành viên

Đã gửi bởi lahantaithe99 on 16-04-2014 - 19:12 trong Góc giao lưu

where? chỉ tui  :ukliam2:  :ukliam2:

Hai bạn gái đứng gần nhất người xem ảnh  :icon6:




#493328 Ảnh thành viên

Đã gửi bởi lahantaithe99 on 16-04-2014 - 19:18 trong Góc giao lưu

chả hiểu j cả. Người nào hàng mấy

Thì 2 bạn đứng gần nhau bên tay phải (so vs người xem), mà có 1 bạn giơ tay chỉ số 2 đó, bạn còn lại đứng ngay bên phải so vs bạn kia




#487485 Ảnh thành viên

Đã gửi bởi lahantaithe99 on 17-03-2014 - 21:07 trong Góc giao lưu

:icon6:




#504121 Ảnh thành viên

Đã gửi bởi lahantaithe99 on 05-06-2014 - 08:58 trong Góc giao lưu

attachicon.gifa.PNG

 

Mình là thằng mặc áo lạnh, người còn lại là tuananh2000 (http://diendantoanho...51-tuananh2000/) :))

Chú Phúc nhìn đệp trai lai láng thế :v  :wub:  :wub:




#504351 Ảnh thành viên

Đã gửi bởi lahantaithe99 on 06-06-2014 - 07:16 trong Góc giao lưu

chú Tế cũng post hình đi chứ

Khổ nỗi anh chả có cái ảnh nào, toàn ảnh thẻ nhìn như thằng trốn tù thôi  :blush:  :ph34r:




#493321 Ảnh thành viên

Đã gửi bởi lahantaithe99 on 16-04-2014 - 18:25 trong Góc giao lưu

Tôi nhìn thấy có 2 bạn gái rất xinh  >:)




#523556 Ảnh thành viên

Đã gửi bởi lahantaithe99 on 08-09-2014 - 21:59 trong Góc giao lưu

này đây :( liệu có được 10 like không ae?

 

1526609_421457907984123_1314193531_n.jpg

Nhìn anh Són mang 1 vẻ đẹp tiềm ẩn và thánh thiện  :closedeyes:  :closedeyes:  >:)  >:)  >:)  >:)  >:)  >:)




#487497 Ảnh thành viên

Đã gửi bởi lahantaithe99 on 17-03-2014 - 21:50 trong Góc giao lưu

mũ đỏ " BOY" đấy

  :icon6:




#486960 Topic về Bất đẳng thức, cực trị THCS

Đã gửi bởi lahantaithe99 on 15-03-2014 - 17:07 trong Bất đẳng thức và cực trị

Cho các số dương a, b, c thỏa mãn a+ b2 + c2 =3

CMR: $\frac{a}{\sqrt{b}}$ + $\frac{b}{\sqrt{c}}$ + $\frac{c}{\sqrt{a}}$  $\geq$  a + b + c

Bài này bạn đã đăng [r bên này rồi mà!

Cách khác

http://diendantoanho...s-cao-xuân-huy/




#483339 Topic về Bất đẳng thức, cực trị THCS

Đã gửi bởi lahantaithe99 on 15-02-2014 - 22:28 trong Bất đẳng thức và cực trị

1)$Cho$$x+y+xy$$=24$. Tìm GTNN$x^{2}+y^{2}$

2)$Cho$$x^2+y^2-xy=4$. TÌm GTLN và GTNN của$x^2+y^2$

1.

$x+y+xy=24\leq x+y+\frac{(x+y)^2}{4}\Rightarrow x+y\geq 8$

$\Rightarrow x^2+y^2\geq \frac{(x+y)^2}{2}\geq \frac{8^2}{2}=32$

2.

$x^2+y^2-xy=4\geq 2xy-xy=xy$

$\Rightarrow x^2+y^2=4+xy\leq 4+4=8$




#494143 $\boxed{\text{Chuyên Đề}}$ Bất đẳng thức - Cực trị

Đã gửi bởi lahantaithe99 on 20-04-2014 - 13:57 trong Bất đẳng thức và cực trị

Xét tích : $(2-a)(2-b)(2-c)\leq 0$ <=> $abc-2(ab+ac+bc)\geq -4$

          Lại có : $(a+b+c)^{2}= a^{2}+b^{2}+c^{2}+2(ab+bc+ac)=9$

Cộng vế ta có : $a^{2}+b^{2}+c^{2}+abc\geq 5$

Mà abc $\geq$ 0 -> đpcm

$a^2+b^2+c^2+abc\geqslant 5$ mà $abc\geqslant 0$ thì không thể suy ra $a^2+b^2+c^2\leqslant 5$ được




#491414 $\boxed{\text{Chuyên Đề}}$ Bất đẳng thức - Cực trị

Đã gửi bởi lahantaithe99 on 08-04-2014 - 17:11 trong Bất đẳng thức và cực trị

Bài 150: CMR với mọi số thực dương $a,b,c$ ta luôn có:

$\sqrt{\frac{a^2+2b^2}{a^2+ab+bc}}+\sqrt{\frac{b^2+2c^2}{b^2+bc+ca}}+\sqrt{\frac{c^2+2a^2}{c^2+ca+ab}}\geq 3$

 

150

 

Áp dụng BĐT Cô si

 

$\sum \sqrt{\frac{a^2+2b^2}{a^2+ab+bc}}\geqslant 3\sqrt[6]{\frac{(a^2+2b^2)(b^2+2c^2)(c^2+2a^2)}{(a^2+ab+bc)(b^2+bc+ac)(c^2+ac+ab)}}$ $(1)$

 

Theo BĐT Bunhiacopxki

 

$(a^2+b^2+b^2)(a^2+c^2+a^2)\geqslant (a^2+ab+bc)^2$

 

Thiết lập tương tự với các số còn lại và rút gọn ta có

 

$(a+2b^2)(b^2+2c^2)(c^2+2a^2)\geqslant (a^2+bc+ab)(b^2+bc+ac)(c^2+ac+ab)$ $(2)$ 

 

Kết hợp $(1)$ với $(2)$ ta có đpcm




#493344 $\boxed{\text{Chuyên Đề}}$ Bất đẳng thức - Cực trị

Đã gửi bởi lahantaithe99 on 16-04-2014 - 20:06 trong Bất đẳng thức và cực trị

Bài143. Cho các số thực dương $x,y,z$ thỏa mãn : $xyz=x+y+z+2$

Chứng minh rằng : $\frac{1}{\sqrt{xy}}+\frac{1}{\sqrt{yz}}+\frac{1}{xz}\leq \frac{3}{2}$

 

Cái chỗ $xz$ kia chắc là $\sqrt{xz}$

 

Từ giả thiết thu được $\frac{x}{x+1}+\frac{y}{y+1}+\frac{z}{z+1}=2$ (cái này tính toán thử ra bao nhiêu TH mới phát hiện ra)

 

Áp dụng BĐT Bunhiacopxki ta có

 

$(\frac{1}{\sqrt{xy}}+\frac{1}{\sqrt{yz}}+\frac{1}{\sqrt{xz}})^2\leqslant (\frac{x}{x+1}+\frac{y}{y+1}+\frac{z}{z+1})(\frac{x+1}{xyz}+\frac{y+1}{xyz}+\frac{z+1}{xyz})$

 

$=\frac{2(x+y+z+3)}{xyz}=\frac{2(xyz+1)}{xyz}=2+\frac{2}{xyz}$

 

Từ giả thiết ta dễ chứng minh $xyz\geqslant 8\Rightarrow 2+\frac{2}{xyz}\leqslant \frac{9}{4}$ 

 

Do đó $(\sum \frac{1}{\sqrt{xy}})^2\leqslant \frac{9}{4}\Rightarrow \sum \frac{1}{\sqrt{xy}}\leqslant \frac{3}{2}$




#493682 $\boxed{\text{Chuyên Đề}}$ Bất đẳng thức - Cực trị

Đã gửi bởi lahantaithe99 on 18-04-2014 - 11:39 trong Bất đẳng thức và cực trị

Học Bdt không giỏi nhưng cũng có nhiều bài hay  :icon6:

145.Cho a,b,c>0 CMR

$\frac{a^{2}}{b}+\frac{b^{2}}{c}+\frac{c^{2}}{a}+a+b+c\geq 2\sqrt{(a^{2}+b^{2}+c^{2})(\frac{a}{b}+\frac{b}{c}+\frac{c}{a})}$

 

Áp dụng BĐT $AM-GM$ ta có

 

$\frac{a^2}{b}+\frac{b^2}{c}+\frac{c^2}{a}+a+b+c\geqslant 2\sqrt{(a+b+c)(\frac{a^2}{b}+\frac{b^2}{c}+\frac{c^2}{a})}$

 

Giờ ta cần chứng minh

 

$(a+b+c)(\frac{a^2}{b}+\frac{b^2}{c}+\frac{c^2}{a})\geqslant (a^2+b^2+c^2)(\frac{a}{b}+\frac{b}{c}+\frac{c}{a})$

 

$\Leftrightarrow a^2+b^2+c^2+\sum \frac{ab^2}{c}\geqslant ab+bc+ac+\sum \frac{a^2b}{c}$

 

$\Leftrightarrow \sum \frac{c(a-b)(a-c)}{b}\geqslant 0$ (cái này đúng theo BĐT S. Chur)

 

Vậy ta có đpcm




#489256 $\boxed{\text{Chuyên Đề}}$ Bất đẳng thức - Cực trị

Đã gửi bởi lahantaithe99 on 28-03-2014 - 20:09 trong Bất đẳng thức và cực trị

124) Cho $S=a^2+b^2+c^2+d^2+ac+bd$ trong đó $ad-bc=1$. Cmr: $S\geq \sqrt{3}$

 

 

 

 

124

Ta có

 

$1+(ac+bd)^2=(ad-bc)^2+(ac+bd)^2=(a^2+b^2)(c^2+d^2)$

 

$\Rightarrow (a^2+b^2)+(c^2+d^2)\geqslant 2\sqrt{(a^2+b^2)(c^2+d^2)}=2\sqrt{1+(ac+bd)^2}$

 

Đậy $ac+bd=x$ thì 

 

$S=a^2+b^2+c^2+d^2+ac+bd\geqslant 2\sqrt{1+x^2}+x$

 

$\Leftrightarrow S^2\geqslant 4(x^2+1)+x^2+4x\sqrt{1+x^2}=(x^2+1)+(2x)^2+4x\sqrt{x^2+1}+3$

 

$=(\sqrt{x^2+1}+2x)^2+3\geqslant 3\Rightarrow S\geqslant \sqrt{3}$

 

p/s: thấy nghi nghi   :wacko:

126

Ta có

 

$\frac{x^2}{y^2}+\frac{y^2}{x^2}+4=(\frac{x}{y}+\frac{y}{x})^2+2=\frac{1}{2}(\frac{x}{y}+\frac{y}{x})^2+\frac{1}{2}(\frac{x}{y}+\frac{y}{x})^2+2$

 

Áp dụng BĐT Cô si thì

 

$\frac{1}{2}(\frac{x}{y}+\frac{y}{x})^2+2\geqslant 2(\frac{x}{y}+\frac{y}{x})(1)$

 

$\frac{1}{2}(\frac{x}{y}+\frac{y}{x})^2\geqslant \frac{1}{2}.2(\frac{x}{y}+\frac{y}{x})=\frac{x}{y}+\frac{y}{x}(2)$

 

Từ $(1)(2)$ suy ra đpcm

 

 




#486039 $\boxed{\text{Chuyên Đề}}$ Bất đẳng thức - Cực trị

Đã gửi bởi lahantaithe99 on 06-03-2014 - 18:44 trong Bất đẳng thức và cực trị

107,  Cho các số thực dương x, y, z thoả mãn $\frac{1}{xy}$   +   $\frac{1}{yz}$   $\frac{1}{zx}$  >   0

Tìm GTNN của biểu thức S  =  $\frac{x^{2}}{yz}$    +    $\frac{y^{2}}{zx}$    +    $\frac{z^{2}}{xy}$

Không biết có phải mk nhầm ko chứ mk nghĩ đk $\sum \frac{1}{xy}>0$ chả cần thiết >:)

Theo bđt S.Vac

$S\geq \frac{(x+y+z)^2}{xy+yz+xz}\geq 3$

(do có bđt quen thuộc là $(x+y+z)^2\geq 3(xy+yz+zx)$




#485492 $\boxed{\text{Chuyên Đề}}$ Bất đẳng thức - Cực trị

Đã gửi bởi lahantaithe99 on 02-03-2014 - 17:27 trong Bất đẳng thức và cực trị

103

$a\sqrt{ac}\leq a\frac{a+c}{2}$$\Rightarrow \sum a\sqrt{ac}\leq \frac{\sum a^{2}+\sum ab}{2}$

lại có $\sum \frac{a^{3}}{b}\sum \frac{a^{4}}{ab}\geq {(\sum a^{2})^{2}}{\sum ab}$

ta cần cm ${(\sum a^{2})^{2}}{\sum ab}\geq \frac{\sum a^{2}+\sum ab}{2}$

mà $\sum a^{2}\geq \sum ab$

$\Rightarrow (\sum a^{2})^{2}\geq (\sum a^{2})(\sum ab)$

$\Rightarrow (\sum a^{2})^{2}\geq (\sum ab)^{2}$

nên ta có đpcm

Chỗ này là sao hả Hoàng????




#484449 $\boxed{\text{Chuyên Đề}}$ Bất đẳng thức - Cực trị

Đã gửi bởi lahantaithe99 on 23-02-2014 - 20:22 trong Bất đẳng thức và cực trị

 

84) Cho $\left\{\begin{matrix}a;b;c>0 & & \\ 3(ab+bc+ca)=1 & & \end{matrix}\right.$. Cmr: $\sum \frac{a}{a^2-bc+1}\geq \frac{1}{a+b+c}$

Lâu ròi ko lên diễn đàn :icon6:

$\sum \frac{a}{a^2-bc+1}=\sum \frac{a}{a^2+3ac+3ab+2bc}=\sum \frac{a^2}{a^3+3a^2c+3a^2b+2abc}$

$\geq \frac{(a+b+c)^2}{\sum a^3+3(a+b)(b+c)(c+a)}$ (áp dụng bđt S.Vac)

$= \frac{(a+b+c)^2}{(a+b+c)^3}=\frac{1}{a+b+c}$




#483495 $\boxed{\text{Chuyên Đề}}$ Bất đẳng thức - Cực trị

Đã gửi bởi lahantaithe99 on 16-02-2014 - 18:26 trong Bất đẳng thức và cực trị

66) Cho $a;b;c$ là 3 cạnh một tam giác có chu vi bằng $6$. Tìm Min $A=3(a^2+b^2+c^2)+2abc$
 

 

Ta có $abc\geq (a+b-c)(a+c-b)(b+c-a)=(6-2a)(6-2b)(6-2b)$

$=216-72(a+b+c)+24(ab+bc+ac)-8abc=24(ab+bc+ac)-216-8abc$

$\Rightarrow 9abc\geq 24(ab+bc+ac)-216\Leftrightarrow 2abc\geq \frac{16}{3}(ab+bc+ac)-48$

$\Rightarrow A\geq 3(a^2+b^2+c^2)+\frac{16}{3}(ab+bc+ac)-48$

$=\frac{a^2+b^2+c^2}{3}+\frac{8}{3}(a+b+c)^2-48$

$\geq 4+96-48=52$




#487948 $\boxed{\text{Chuyên Đề}}$ Bất đẳng thức - Cực trị

Đã gửi bởi lahantaithe99 on 20-03-2014 - 17:03 trong Bất đẳng thức và cực trị

 

Bài 111: Cho $a,b,c$ là các số thực dương. CMR:

$\sum \frac{a}{(b+c)^2}\geq \frac{9}{4(\sum a)}$

 

P/s: HẾT!!!  :rolleyes: 

111.

BĐT cần chứng minh tương đương

 

$(a+b+c)(\frac{a}{(b+c)^2}+\frac{b}{(c+a)^2}+\frac{c}{(a+b)^2})\geqslant \frac{9}{4}$

 

Áp dụng bđt Bunhiacopxki

 

$(a+b+c)(\frac{a}{(b+c)^2}+\frac{b}{(c+a)^2}+\frac{c}{(a+b)^2})\geqslant (\frac{a}{b+c}+\frac{b}{c+a}+\frac{c}{a+b})^2$

$\geqslant (\frac{3}{2})^2=\frac{9}{4}$ (đúng theo BĐT Nesbit)




#488157 $\boxed{\text{Chuyên Đề}}$ Bất đẳng thức - Cực trị

Đã gửi bởi lahantaithe99 on 21-03-2014 - 21:24 trong Bất đẳng thức và cực trị

Mọi người giúp nhanh cho mình nha thanks nhìu  :(

114. Cho $x,y,z>0$ và $x+y+z=1$. Tính GTLN $P=\frac{1}{2x+y+z}+\frac{1}{x+2y+z}+\frac{1}{x+y+2z}$

 

115. Cho $x,y,z>0$ và $x+y+z=1$. Tính GTLN $s=\frac{x}{x+1}+\frac{y}{y+1}+\frac{z}{z+1}$

 

116. Cho $a,b,c>1$. CMR: $\frac{4a^{2}}{a-1}+\frac{5b^{2}}{b-1}+\frac{3c^{2}}{c-1}\geq 48$

 

117.Cho $a,b,c>0$. CMR $\frac{a^{3}}{b}+\frac{b^{3}}{c}+\frac{c^{3}}{a}\geq ab+bc+ca$

 

118. Cho $x,y>0$ và $x+y=1$. CMR $P=\frac{1}{x^{3}+y^{3}}+\frac{1}{xy}\geq 4+2\sqrt{3}$

Đã có tất trong đây

http://diendantoanho...1x2yzfrac1xy2z/




#489682 $\boxed{\text{Chuyên Đề}}$ Bất đẳng thức - Cực trị

Đã gửi bởi lahantaithe99 on 30-03-2014 - 19:04 trong Bất đẳng thức và cực trị

  $a,b > 0$  thỏa   mãn    $3a^{2}  + 2b^{2} \leq  5$

Tìm min     $ \frac{a}{2}+ b+  \frac{2}{ab}$

:nav:  :nav:  :nav:

Áp dụng BĐT Cô si 

 

$5=3a^2+2b^2=a^2+a^2+a^2+b^2+b^2\geqslant 5.\sqrt[5]{a^6b^4}\Rightarrow a^6b^4\leqslant 1\Rightarrow a^3b^2\leqslant 1$

 

Tiếp tục áp dụng BĐT cô si và sử dụng $a^3b^2\leqslant 1$

 

$\frac{a}{2}+b+\frac{2}{ab}=\frac{a}{2}+\frac{b}{2}+\frac{b}{2}+\frac{1}{2ab}+\frac{1}{2ab}+\frac{1}{2ab}+\frac{1}{2ab}$

 

$\geqslant 7.\sqrt[7]{\frac{1}{2^7a^3b^2}}\geqslant \frac{7}{2}$

 

@Viet Hoang 99: Đừng sửa trích dẫn của người khác nhé.




#482960 $\boxed{\text{Chuyên Đề}}$ Bất đẳng thức - Cực trị

Đã gửi bởi lahantaithe99 on 13-02-2014 - 20:49 trong Bất đẳng thức và cực trị

 

60) Cho $a;b;c$ là độ dài 3 cạnh một tam giác. Cmr: $\sum \frac{a}{\sqrt{b+c-a}}\geq \sum \sqrt{a}$

 

 

 

P/s: Anh Daicagiangho1998 học KHTN nên cứ từ từ mà làm thôi, chứ có đề phát đã ăn hết sạch luôn vậy :D

Đặt $x^2=b+c-a;y^2=a+c-b;z^2=a+b-c$

$\Rightarrow a=\frac{y^2+z^2}{2};b=\frac{x^2+z^2}{2};c=\frac{x^2+y^2}{2}$

BĐT cần cm trở thành $\sum \frac{x^2+y^2}{2x}\geq \sum \sqrt{\frac{x^2+y^2}{2}}$

Áp dụng bđt Cauchy-Shwarz có

$\sum \frac{x^2+y^2}{2x}\geq \frac{(\sum\sqrt{\frac{x^2+y^2}{2}})^2 }{x+y+z}$

Bằng $AM-GM$ ta dễ chứng minh $\sum x\leq \sum \sqrt{\frac{x^2+y^2}{2}}$

$\Rightarrow \sum \frac{x^2+y^2}{2x}\geq \sum\sqrt{\frac{x^2+y^2}{2}}$ (đpcm)




#488715 $\boxed{\text{Chuyên Đề}}$ Bất đẳng thức - Cực trị

Đã gửi bởi lahantaithe99 on 25-03-2014 - 17:54 trong Bất đẳng thức và cực trị

Mình vẫn chưa hiểu chỗ này suy ra từ đâu

Ta có bdt quen thuộc $ab\leqslant \frac{(a+b)^2}{4}$

Trong bài này $2xy$ đóng vai trò là $a$ còn $x^2+y^2$ đóng vai trò là $b$




#491386 $\boxed{\text{Chuyên Đề}}$ Bất đẳng thức - Cực trị

Đã gửi bởi lahantaithe99 on 08-04-2014 - 12:58 trong Bất đẳng thức và cực trị

 

Bài 151: Cho $a,b,c$ là các số thực dương thỏa mãn $a+b+c=3$. CMR:

$\sqrt{\frac{a+b}{c+ab}}+\sqrt{\frac{b+c}{a+bc}}+\sqrt{\frac{c+a}{b+ca}}\geq 3$

 

151

Áp dụng BĐT Cô si

 

$\sum \sqrt{\frac{a+b}{c+ab}}\geqslant 3.\sqrt[6]{\frac{(a+b)(b+c)(c+a)}{(c+ab)(b+ca)(a+bc)}}$ $(1)$

 

Ta có $(a+bc)(b+ac)\leqslant (\frac{a+bc+b+ac}{2})^2=\frac{(a+b)^2(c+1)^2}{4}$

 

CMTT và rút gọn thì $(a+bc)(b+ac)(c+ab)\leqslant \frac{(a+b)(b+c)(c+a)(a+1)(b+1)(c+1)}{8}$

 

Mà $\frac{(a+1)(b+1)(c+1)}{8}\leqslant \frac{(a+b+c+3)^3}{27.8}=1$

 

suy ra $(a+bc)(b+ac)(c+ab)\leqslant (a+b)(b+c)(c+a)(2)$

 

Kết hợp $(1)$ và $(2)$ ta có đpcm